Mulitvariable Calculus Four Dimensional Sphere

  • Thread starter Thread starter Girth
  • Start date Start date
  • Tags Tags
    Calculus Sphere
Click For Summary
The discussion centers on calculating the 4-dimensional volume of a ball defined by the equation x^2 + y^2 + z^2 + w^2 = R^2 using double polar coordinates. The user successfully transformed the equation into r^2 + s^2 = R^2, applying the identity for sine and cosine. However, they express confusion about the next steps in setting up the integral for volume calculation. Suggestions are sought on how to proceed with the integration process, drawing parallels to the method used for 3D spheres. The conversation highlights the need for guidance in transitioning from the equation simplification to the volume integral setup.
Girth
Messages
1
Reaction score
0

Homework Statement


Use double polar coordinates x=rcosθ y=rsinθ z=scosφ w=ssinφ in R^4
to compute the 4-dimensional volume of the ball x^2 +y^2 +z^2+w^2 = R^22. The attempt at a solution
I first substituted the polar coordinates into the given equation getting: r^2cos^2(θ)+r^2sin^2(θ)+s^2cos^2(φ)+r^2sin^2(φ) = R^2
this simplified to r^2 + s^2 = R^2 using the identity: cos^2(θ)+sin^2(θ)=1
After this I'm confused with where to go. Any tips would help, thanks.
 
Last edited:
Physics news on Phys.org
Girth said:

Homework Statement


Use double polar coordinates x=rcosθ y=rsinθ z=scosφ w=ssinφ in R^4
to compute the 4-dimensional volume of the ball x^2 +y^2 +z^2+w^2 = R^2


2. The attempt at a solution
I first substituted the polar coordinates into the given equation getting: r^2cos^2(θ)+r^2sin^2(θ)+s^2cos^2(φ)+r^2sin^2(φ) = R^2
this simplified to r^2 + s^2 = R^2 using the identity: cos^2(θ)+sin^2(θ)=1
After this I'm confused with where to go. Any tips would help, thanks.

To find the volume I think you must setup a 4D integral similar to how one finds the volume of a 3D sphere:

http://en.wikipedia.org/wiki/N-sphere
 
Question: A clock's minute hand has length 4 and its hour hand has length 3. What is the distance between the tips at the moment when it is increasing most rapidly?(Putnam Exam Question) Answer: Making assumption that both the hands moves at constant angular velocities, the answer is ## \sqrt{7} .## But don't you think this assumption is somewhat doubtful and wrong?

Similar threads

  • · Replies 2 ·
Replies
2
Views
2K
  • · Replies 3 ·
Replies
3
Views
3K
  • · Replies 3 ·
Replies
3
Views
1K
  • · Replies 21 ·
Replies
21
Views
3K
  • · Replies 4 ·
Replies
4
Views
2K
  • · Replies 13 ·
Replies
13
Views
3K
Replies
3
Views
2K
  • · Replies 3 ·
Replies
3
Views
3K
  • · Replies 3 ·
Replies
3
Views
2K
Replies
6
Views
2K